Answer:Geometric Sequence: an=a1rn−1. Explanation: a6=7×2(6−1)=7×32=224. hope that helped.
Step-by-step explanation:
Please take a look at the picture
Answer:
B) [tex]-\frac{a}{-b}[/tex]
Step-by-step explanation:
The value [tex]-\frac{-a}{b}[/tex] is the same as [tex]\frac{a}{b}[/tex] since the negatives cancel out to make a positive.
Option A is incorrect since the overall value of the term would be negative. Option B however is correct since there will not be a negative value in the denominator of a fraction and thus it cancels out with the negative in front of the fraction to produce the value of [tex]\frac{a}{b}[/tex] which is the same as the expression given in the question, thus it is option B.
PLEASE HELP!!
Guys please help i need this
Answer:
Because they are both base angles of isosceles triangles
Subtract.
7 1/5−(−3/5)
Enter your answer in the box as mixed number in simplest form.
Answer:
7 4/5
Step-by-step explanation:
when you subtract, keep the first value then change subtract to addition and, finally, change the sign of the second value
this problem, 7 1/5 -(-3/5) is the same as this addition problem:
7 1/5 + 3/5
this equals 7 4/5
1. Identify the place value for the following: 162.891
Answer:
Hundreds : 1
Tens : 6
Ones : 2
.
Tenths : 8
Hundredths : 9
Thousandths : 1
A restaurant has an electronic system that randomly selects customers when they pay for their meal to receive a coupon for their next visit. Each customer has a 10\%10%10, percent probability of being selected to receive a coupon, and one customer being selected or not doesn't affect whether or not another customer will be selected.
Suppose that 222 customers pay for their meals, one after the other.
What is the probability that NEITHER customer is selected to receive a coupon?
Round your answer to two decimal places.
Answer:
Step-by-step explanation:
Using the binomial distribution, it is found that there is a 0.81 = 81% probability that NEITHER customer is selected to receive a coupon.
For each customer, there are only two possible outcomes, either they receive the coupon, or they do not. The probability of a customer receiving the coupon is independent of any other customer, which means that the binomial distribution is used to solve this question.
Binomial probability distribution
0.81 = 81% probability that NEITHER customer is selected to receive a coupon.
What is the better buy?
16 oz of goldfish for $4.75
9 oz of goldfish for $2.50
Answer:
Maybe 9 oz, if I’m wrong sorry! D:
On October 1, Organic Farming purchases wind turbines for $140,000. The wind turbines are expected to last six years, have a salvage
value of $20,000, and be depreciated using the straight-line method.
1. Compute depreciation expense for the last three months of the first year.
2. Compute depreciation expense for the second year.
9514 1404 393
Answer:
$5000$20,000Step-by-step explanation:
1.The annual depreciation is the predicted change in value each year. The change in value for 6 years is predicted to be ...
$140,000 -20,000 = $120,000 . . . . loss of value in 6 years
3 months is 1/4 of one year, so the loss of value in that time period is ...
$120,000/6 × 1/4 = $5000
The depreciation expense for the last 3 months of the first year is $5000.
__
2.The depreciation expense for the second year is ...
$120,000/6 = $20,000
consider the graph of a linear equation. see below , which statement is true?
Answer:
B
Step-by-step explanation:
If you go to -4, 0 the line intersects that point
Needing to find the answer to this question attached. Thank you
[tex]\sin \theta -1 =0\\\\\ \implies \sin \theta = 1\\\\ \implies \theta = \dfrac{\pi}2 + 2\pi n\\\\\text{For n =0,}\\\\\theta = \dfrac{\pi}2[/tex]
$32.00for a 14 2/9 km taxi ride. What is the cost per kilometer
[tex]\begin{array}{ccll} \$&km\\ \cline{1-2} 32&14\frac{2}{9}\\[1em] x&1 \end{array}\implies \cfrac{32}{x}=\cfrac{14\frac{2}{9}}{1}\implies \cfrac{32}{x}=14\frac{2}{9}\implies \cfrac{32}{x}=\cfrac{14\cdot 9+2}{9} \\\\\\ \cfrac{32}{x}=\cfrac{128}{9}\implies 288=128x\implies \cfrac{288}{128}=x\implies \stackrel{~\hfill \textit{2 bucks and 25 cents}}{\cfrac{9}{4}=x\implies 2\frac{1}{4}}=x[/tex]
Elizabeth recently bought a new bike, and she has started biking to and from work. The
round-trip distance is 9 kilometers.
Write an equation that shows the relationship between the number of round trips, x, and
the total number of kilometers Elizabeth has biked, y.
Answer:
the answer is 9x
Step-by-step explanation:
y=9x is the relationship between the number of round trips, x, and the total number of kilometers Elizabeth has biked, y.
What is Equation?Two or more expressions with an Equal sign is called as Equation.
Given that,
Elizabeth recently bought a new bike, and she has started biking to and from work.
The round-trip distance is 9 kilometers.
We need to find an equation that shows the relationship between the number of round trips, x, and the total number of kilometers Elizabeth has biked, y.
y=9x
Hence y=9x is the relationship between the number of round trips, x, and the total number of kilometers Elizabeth has biked, y.
To learn more on Equation:
https://brainly.com/question/10413253
#SPJ2
please hurry this is timed and I only have 20 mins left!!!
Answer:
x = 6 2/3 The supplementary pair of angle is 78 degree and z = 112degree = 180 degree First step is where we can also see z its opposite angle is vertical opposite angle to 112 degree given and label as vertical angle - where we see a line through the 180 degree line at any angle we can call the pair supplementary- Pairs OR sets of angles an only be called consecutive when they are grouped with a set of parallel line across a straight line and it is only because they add up to 180 they could be either terms so as there isn't parallel shown ; we know we an call a pair of angles upon a straight line supplementary.angles.
Step-by-step explanation:
6 x +38 (+112) = 180 degree as angles on a straight line add up to 180 6 x +38 (+112) = 180 6 x +140 = 180 6 x + 140- 140 = 180-140 6 x = 40 x = 40/6 x = 6.66666667 x = 6 2/3 Proof (6( 6 2/3)) + 38 + 112 = 180 = (40+38)+112 = 180 = 78 +112 = 180 Proves the supplementary angle is 78 degree.
what is 4% of 32? is it a percent, base, or amount
Answer:
1.28 - amount
Step-by-step explanation:
You would multiply 32 times 0.04 and get 1.28 which I assume would be an amount.
The amount of 4% of 32 is 1.28.
We have to determine what is 4% of 32.
What is the percentage?
This free percentage calculator computes a number of values involving percentages, including the percentage difference between two given values.
Suppose it is an x percent.
Therefore x multiply 32 times 0.04 and get 1.28 an amount.
To learn more about the percentage of visits:
https://brainly.com/question/24304697
#SPJ2
-36.12 divided by -5 3/5
- BRAINLIEST answerer
rewrite the expression 44 + 32 + 112 using the distributive property
Answer:
4(11+8+28)
Step-by-step explanation:
44 + 32 + 112
4(11+8+28)
I found the GCF of all three numbers.
Hope this helps :)
Which of the following is the greatest common factor of 48 and 32?
Answer:
8
Step-by-step explanation:
cus...
In what time will $1000 amount to $1331 at 10% per annum, compounded annually?
3 Years
Step-by-step explanation:Compound interest is a multiplier that affects the new value each time.
If the compound interest is 10% per annum, the multiplier each year is 1.1.
Year 1: $1000 * 1.1 = $1100
Year 2: $1100* 1.1 = $1210
Year 3: $1210* 1.1 = $1331
This means that it will take 3 years for $1000 to amount to $1331 at 10% per annum.
Alternatively, you can use a logarithm. The calculation can be written as:
1000 * [tex]1.1^n[/tex] = 1331, where n is the number of years.
Rearranged, this gives [tex]1.1^n[/tex] = 1331/1000 or [tex]1.1^n[/tex] = 1.331.
A logarithm can work out the value of n, and is written in the form log1.1(1.331), which gives a value of 3.
plz help very easy i pormise
Answer:
8/90
Step-by-step explanation:
hope this helps
Answer:
It's 38/45
Step-by-step explanation:
I don't really have a step-by-step explanation. I just looked it up! ;) good luck
1. Andrea’s car has broken down for the third time this month, and she has decided to buy a different car. Her dream car, a 2016 Chevrolet Camaro, costs $38,000. She has a $4,000 down payment saved, and the bank is offering 3.00% APR on 60 month loans for new cars.
a. How much will Andrea need to borrow to buy the Camaro?
b. What would Andrea need to type in Excel to figure her monthly payment, and what will her monthly payment be?
c. What will the total of all of Andrea’s payments be over the lifetime of the car loan?
d. How much will Andrea pay in interest over the lifetime of the car loan?
A) Andrea will need to borrow $34,000 to buy the Camaro
B) Andrea will need to type =(34000x1.03 ^ 5) / 60 in Excel to figure her monthly payment, which will be $656,92.
C) The total of all of Andrea’s payments be over the lifetime of the car loan will be $39.415,31
D) Andrea will pay $5415.31 in interest over the lifetime of the car loan
Since Andrea's car has broken down for the third time this month, and she has decided to buy a different car, and her dream car, a 2016 Chevrolet Camaro, costs $ 38,000, and she has a $ 4,000 down payment saved, and the bank is offering 3.00% APR on 60 month loans for new cars, to determine A) how much will Andrea need to borrow to buy the Camaro; B) what would Andrea need to type in Excel to figure her monthly payment from her, and what will her her monthly payment be from her; C) what will the total of all of Andrea’s payments be over the lifetime of the car loan; and D) how much will Andrea pay in interest over the lifetime of the car loan, the following calculations must be performed:
A) 38000 - 4000 = X 34000 = X
B) = (34000x1.03 ^ 5) / 6039.415,31 / 60656,92C) 34000 x 1.03^5 = X34000 x 1.1592 = X39.415,31 = XD) (34000 x 1.03^5) - 34000 = X(34000 x 1.1592) - 34000 = X39415.31 - 34000 = X5415.31 = X
Learn more in https://brainly.com/question/14295570
What’s the slope of 10,6 and 4, 1.2
let's firstly change the 1.2 to a fraction
[tex]1.\underline{2}\implies \cfrac{12}{1\underline{0}}\implies \cfrac{6}{5} \\\\[-0.35em] ~\dotfill\\\\ (\stackrel{x_1}{10}~,~\stackrel{y_1}{6})\qquad (\stackrel{x_2}{4}~,~\stackrel{y_2}{\frac{6}{5}}) \\\\\\ \stackrel{slope}{m}\implies \cfrac{\stackrel{rise} {\stackrel{y_2}{\frac{6}{5}}-\stackrel{y1}{6}}}{\underset{run} {\underset{x_2}{4}-\underset{x_1}{10}}}\implies \cfrac{~~ \frac{6-30}{5}~~}{-6}\implies \cfrac{~~ \frac{-24}{5}~~}{-6}\implies \cfrac{~~ -\frac{24}{5}~~}{-\frac{6}{1}}[/tex]
[tex]-\cfrac{\stackrel{4}{~~\begin{matrix} 24 \\[-0.7em]\cline{1-1}\\[-5pt]\end{matrix}~~}}{5}\cdot -\cfrac{1}{\underset{1}{~~\begin{matrix} 6 \\[-0.7em]\cline{1-1}\\[-5pt]\end{matrix}~~}}\implies \boxed{\cfrac{4}{5}}[/tex]
Please help! (Urgent)
Only answer if correct.
I will be asking more so stay tuned.
Answer:
Number of rotations: 5
Slope: 1/2
Step-by-step explanation:
Answer: why did you ask twice for the same problem
Step-by-step explanation:
Which graph represents 6x-2y>-11
Answer:
Answer:
C
Step-by-step explanation:
Remember when using the inequality sign < or > the line will be dotted so we can immediately eliminate answer choices b and d
Now let us solve for y
6x-2y>-11
step 1 subtract each side by 6x
now we have
-2y>-11-6x
step 2 divide each side by -2
-11-/2=11/2
-6/-2=3
IMPORTANT -when you divide inequalities by a negative number you flip the inequality sign
So now we have
See image below for reference
When the inequality is facing x the solutions are going to be on top of the line
hope this helps!
Step-by-step explanation:
How would I write 129.701 in expanded form
solve using quadratic formula
6x^2-x=2
[tex]6x^2-x=2\implies 6x^2-1x-2=0 \\\\[-0.35em] ~\dotfill\\\\ ~~~~~~~~~~~~\textit{quadratic formula} \\\\ \stackrel{\stackrel{a}{\downarrow }}{6}x^2\stackrel{\stackrel{b}{\downarrow }}{-1}x\stackrel{\stackrel{c}{\downarrow }}{-2}=0 \qquad \qquad x= \cfrac{ - b \pm \sqrt { b^2 -4 a c}}{2 a}[/tex]
[tex]x = \cfrac{ -(-1) \pm \sqrt { (-1)^2 -4(6)(-2)}}{2(6)}\implies x = \cfrac{1\pm\sqrt{1+48}}{12} \\\\\\ x = \cfrac{1\pm\sqrt{49}}{12}\implies x = \cfrac{1\pm 7}{12}\implies x = \begin{cases} \frac{8}{12}\to &\frac{2}{3}\\[1em] -\frac{6}{12}\to &-\frac{1}{2} \end{cases}[/tex]
There are integers that are not rational
numbers, true or false?
Answer:
integers are always rational. They don't include decimals
Step-by-step explanation:
Answer:
False
Step-by-step explanation:
All integers are rational numbers, since they can be expressed as a ratio (fraction) with a denominator of 1.
Will the answer to the addition problem below be odd or even?
2156 +992 = ?
Answer:
Even
Step-by-step explanation:
When two even numbers are added there will be an even result. To check this adding 2156 and 992 gets 3148 which is an even number.
which graph be represents the soulution set of y[tex]\leq[/tex]3/4x-4?
2-2 this is a freebie for points guys
Answer:
2-2=0
Step-by-step explanation:
You have 2 cookies and you give it to 2 friends.
Answer:
0
Step-by-step explanation:
According to the Laws of Physics, yeah I don't wanna try explaining this
The school football team is hosting a fundraiser dinner. Tickets are $20 for adults, $15 for seniors, and $6 for children. The expression 20a + 15s + 6c represents the total sales for a adult, s senior tickets, and c child tickets. Mariel sold 6 adult, 4 senior, and 8 child tickets.
20a + 15s + 6c = 20 (6) + 15 (4) + 6 (8) = ? + ? + ?
amara needs to create a special tasting menu at her restaurant. She needs to select 4 dishes from 7 available dishes and put them in a tasty sequence. How many unique ways are there to arrange 4 of the 7 dishes?
By taking in mind the number of combinations of 4 out of the 7 dishes, and the possible orders of these 4 dishes, we will find that there are 840 unique ways to arrange 4 of the 7 dishes.
We know that if we have a set of N elements, the total number of different combinations of K elements (K ≤ N) out of the N elements is given by:
[tex]C(N, K) = \frac{N!}{(N - K)!*K!}[/tex]
In this particular case, we have:
N = number of available dishes = 7K = number that we need to select = 4Then the number of different combinations is given by:
[tex]C(7, 4) = \frac{7!}{(7 - 4)!*4!} = \frac{7*6*5}{3*2} = 35[/tex]
Now we have 4 dishes selected, but we also need to order them.
For the first dish, there are 4 options.For the second, there are 3 options (as one was already selected).For the third, there are 2 options.For the last one there is only one option.The number of different arrangements of the dishes is given by the product between the numbers of options above, this gives:
4*3*2*1 = 24
Then the total number of unique ways of arranging 4 of the 7 dishes is:
C = 24*35 = 840
Where we take in mind the possible number of combinations of 4 out of the 7 dishes, and the possible orders of these 4 dishes.
If you want to learn more, you can read:
https://brainly.com/question/9976085